Binet-Cauchy 公式及其应用

SalomeJLQ

2023-08-26 22:49:00

Theory

Binet-Cauchy 公式广泛运用于与乘积矩阵相关的秩、子式和行列式等计算和分析中,并能够证明一些重要且著名的恒等式。本文详细介绍了 Binet-Cauchy 公式的来路、证明,并就其一些基础且重要的应用做了较为全面的介绍。 ## 准备:行列式的 $k$ 阶子式 所谓 $\boldsymbol k$ **阶子式**是指选定 $\boldsymbol A$ 的 $k$ 行 $i_1,\cdots,i_k$ 与 $k$ 列 $j_1,\cdots,j_k$,由着 $k$ 行 $k$ 列相交的元素组成的方阵的行列式,记作 $$ \boldsymbol A\begin{pmatrix}i_1,i_2,\cdots,i_k\\j_1,j_2,\cdots,j_k\end{pmatrix}:=\begin{vmatrix} a_{i_1j_1}&a_{i_1j_2}&\cdots&a_{i_1j_k}\\ a_{i_2j_1}&a_{i_2j_2}&\cdots&a_{i_2j_k}\\ \vdots&\vdots&\ddots&\vdots\\ a_{i_kj_1}&a_{i_kj_2}&\cdots&a_{i_kj_k} \end{vmatrix}, $$ 而其对应的**余子式**即为将 $\boldsymbol A$ 划去这 $k$ 行 $k$ 列后组成的矩阵的行列式。于是设余下的行与列为 $i'_1,\cdots,i'_{n-k}$ 及 $j'_1,\cdots,j'_{n-k}$。显然余子式是一个 $n-k$ 阶子式,它等于 $$ \boldsymbol A\begin{pmatrix}i'_1,i'_2,\cdots,i'_{n-k}\\j'_1,j'_2,\cdots,j'_{n-k}\end{pmatrix}, $$ 所说的 $k$ 阶子式的**代数余子式**则是 $$ (-1)^{(i_1+\cdots+i_k)+(j_1+\cdots+j_k)}\boldsymbol A\begin{pmatrix}i'_1,i'_2,\cdots,i'_{n-k}\\j'_1,j'_2,\cdots,j'_{n-k}\end{pmatrix}. $$ 可自行对照 $1$ 阶子式的代数余子式,它是我们早先已熟知的。需要注意,这里 $\{i\},\,\{j\},\,\{i'\},\,\{j'\}$ 都是没有顺序的,等价地,它们皆从小到大排列。 ### 主子式 矩阵 $\boldsymbol A$ 的一个 $ r$ 阶**主子式**是它的一个行指标与列指标相同的 $r$ 阶子式 $$ \boldsymbol A\begin{pmatrix} i_1,i_2,\cdots,i_r\\ i_1,i_2,\cdots,i_r \end{pmatrix}. $$ # Binet-Cauchy 公式 Binet-Cauchy 公式是描述矩阵乘积的行列式的一个相当重要的公式。我们先给出这个公式。 $\bf Theorem\;1.\;(Binet\text -Cauchy)\quad$设 $\boldsymbol{A}=(a_{ij})_{s\times n},\,\boldsymbol{B}=(b_{ij})_{n\times s}$,则有: - 如果 $s>n$,则 $|\boldsymbol{AB}|=0$。 - 如果 $s\leqslant n$,那么: $$ |\boldsymbol{AB}|=\sum_{1\leqslant v_1<\cdots<v_s\leqslant n}\boldsymbol A\begin{pmatrix} 1,2,\cdots,s\\v_1,v_2,\cdots,v_s \end{pmatrix} \boldsymbol B\begin{pmatrix} v_1,v_2,\cdots,v_s\\1,2,\cdots,s \end{pmatrix} $$ 为了证明 Binet-Cauchy 公式,我们先来引入 Laplace 定理。 ## Laplace 定理 Laplace 定理是行列式的 $k$ 行展开定理,它表明任选 $\boldsymbol A$ 的 $k$ 行,则 $\det\boldsymbol A$ 为由此产生的所有 $k$ 阶子式与其对应 $k$ 阶代数余子式之积的和,亦即如下定理。 $\mathbf{Theorem\;2.\;(Laplace)}\quad$对任意给定的 $k$ 行 $i_1,i_2,\cdots,i_k$,以下等式成立: $$ |{\boldsymbol A}|=\sum_{1<j_1\leqslant\dots<j_k\leqslant n}\boldsymbol A\begin{pmatrix}i_1,i_2,\cdots,i_k\\j_1,j_2,\cdots,j_k\end{pmatrix}(-1)^{\left(\sum_{x}i_x\right)+\left(\sum_{x}j_x\right)}\boldsymbol A\begin{pmatrix}i'_1,i'_2,\cdots,i'_{n-k}\\j'_1,j'_2,\cdots,j'_{n-k}\end{pmatrix} $$ ### Laplace 定理的证明 考虑在 $|\boldsymbol A|$ 的表达式中,将 $i_1,\cdots,i_k$ 放在前 $k$ 个位置。由此,我们首先在排列中任意取出 $j_1,\cdots,j_k$ 个数,并将其重排为 $\mu_1,\cdots,\mu_k$,以此与 $i_1,\cdots i_k$ 匹配;然后对于余下的 $n-k$ 个数重排,与 $i'_1,\cdots,i'_{n-k}$ 这些行匹配。从组合意义讲,这样能够取遍所有 $[n]$ 的置换。事实上其方案数为 $$ \binom{n}{k}\cdot k!\cdot(n-k)!=n!\;, $$ 于是这样确实能够取遍 $[n]$ 的所有置换。由此,我们确实可以按这种方式书写行列式,即 $$ \begin{aligned} |\boldsymbol A|&=\sum_{\mu_1\cdots\mu_kv_1\cdots v_{n-k}}(-1)^{\tau(i)+\tau(j)}a_{i_1\mu_1}\cdots a_{i_k\mu_k}a_{i'_1v_1}\cdots a_{i'_{n-k}v_{n-k}}\\ &=\sum_{1<j_1\leqslant\dots<j_k\leqslant n}\sum_{(\mu_1\cdots\mu_k)\in\sigma(j_1,\cdots,j_k)}\sum_{v_1\cdots v_{n-k}}a_{i_1\mu_1}\cdots a_{i_k\mu_k}a_{i'_1v_1}\cdots a_{i'_{n-k}v_{n-k}} \,, \end{aligned} $$ 其中 $\tau(i)+\tau(j)=\tau(i_1\cdots i_ki'_1\cdots i'_{n-k})+\tau(\mu_1\cdots\mu_kv_1\cdots v_{n-k})$。根据我们前述生成排列的方法,可得 $$ \begin{aligned} \tau(i_1&\cdots i_ki'_1\cdots i'_{n-k})\\ &=(i_1-1)+(i_2-2)+\cdots+(i_k-k)\\ &=(i_1+\cdots+i_k)-\frac{k(k+1)}{2}. \end{aligned} $$ 而根据逆序对的性质,设 $j_1,\cdots,j_k$ 经过 $s$ 次互换后变为 $\mu_1,\cdots,\mu_k$,我们又得到 $$ \begin{aligned} (-1)&^{\tau(\mu_1\cdots\mu_kv_1\cdots v_{n-k})}\\ &\;=(-1)^s(-1)^{\tau(j_1\cdots j_kv_1\cdots v_{n-k})}\\ &\;=(-1)^{\tau(\mu_1\cdots\mu_k)}(-1)^{(i_1-1)+(i_2-2)+\cdots+(i_k-k)+\tau(v_1\cdots v_{n-k})}\\ &\;=(-1)^{(j_1+j_2+\cdots+j_k)-\frac{1}{2}k(k+1)}(-1)^{\tau(\mu_1\mu_2\cdots\mu_k)}(-1)^{\tau(v_1\cdots v_{n-k})}. \end{aligned} $$ 由此可得 $$ \begin{aligned} |\boldsymbol A| &=\sum_{1<j_1\leqslant\dots<j_k\leqslant n}\sum_{\mu_1\cdots\mu_k}\sum_{v_1\cdots v_{n-k}}(-1)^{\tau(i)+\tau(j)}\left(\prod_{p\leqslant k}a_{i_p\mu_p}\right)\!\left(\prod_{q\leqslant n-k}a_{i'_qv'_q}\right)\\ &=\sum_{1<j_1\leqslant\dots<j_k\leqslant n}(-1)^{(i_1+\cdots+i_k)-\frac{1}{2}k(k+1)}(-1)^{(j_1+j_2+\cdots+j_k)-\frac{1}{2}k(k+1)}\\&\qquad\quad\sum_{\mu_1\cdots\mu_k}\sum_{v_1\cdots v_{n-k}}(-1)^{\tau(\mu_1\mu_2\cdots\mu_k)}(-1)^{\tau(v_1\cdots v_{n-k})}\left(\prod_{p\leqslant k}a_{i_p\mu_p}\right)\!\left(\prod_{q\leqslant n-k}a_{i'_qv'_q}\right)\\ &=\sum_{1<j_1\leqslant\dots<j_k\leqslant n}(-1)^{(i_1+\cdots+i_k)+(j_1+j_2+\cdots+j_k)} \\&\qquad\qquad\quad\times\left(\sum_{\mu_1\cdots\mu_k}(-1)^{\tau(\mu_1\mu_2\cdots\mu_k)}\prod_{p\leqslant k}a_{i_p\mu_p}\right)\\ &\qquad\qquad\qquad\qquad\quad\times\left(\sum_{v_1\cdots v_k}(-1)^{\tau(v_1v_2\cdots v_k)}\prod_{q\leqslant n-k}a_{i'_qv_q}\right)\\ &=\sum_{1<j_1\leqslant\dots<j_k\leqslant n}\boldsymbol A\begin{pmatrix}i_1,i_2,\cdots,i_k\\j_1,j_2,\cdots,j_k\end{pmatrix}(-1)^{\left(\sum_{x}i_x\right)+\left(\sum_{x}j_x\right)}\boldsymbol A\begin{pmatrix}i'_1,i'_2,\cdots,i'_{n-k}\\j'_1,j'_2,\cdots,j'_{n-k}\end{pmatrix}. \end{aligned} $$ ### Laplace 定理的应用 以下推论是与证明 Binet-Cauchy 公式有关的 Laplace 定理的最重要的结论。 $\mathbf{Corollary\;3.}$ $$ \begin{vmatrix} a_{11}&\cdots&a_{1k}&0&\cdots&0\\ \vdots&\ddots&\vdots&\vdots&\ddots&\vdots\\ a_{k1}&\cdots&a_{kk}&0&\cdots&0\\ c_{11}&\cdots&c_{1k}&b_{11}&\cdots&b_{1r}\\ \vdots&\ddots&\vdots&\vdots&\ddots&\vdots\\ c_{r1}&\cdots&c_{rk}&b_{r1}&\cdots&b_{rr}\\ \end{vmatrix}= \begin{vmatrix} a_{11}&\cdots&a_{1k}\\ \vdots&\ddots&\vdots\\ a_{k1}&\cdots&a_{kk} \end{vmatrix}{\;\cdot\;} \begin{vmatrix} b_{11}&\cdots&b_{1r}\\ \vdots&\ddots&\vdots\\ b_{r1}&\cdots&b_{rr} \end{vmatrix} $$ $Proof.\quad$令 $i_1,i_2,\cdots,i_k$ 为 $1,2,\cdots,k$ 即可。 它可以简略地记作 $$ \begin{vmatrix} \boldsymbol A&\boldsymbol 0\\ \boldsymbol C&\boldsymbol B \end{vmatrix}=\left|\boldsymbol A\right|\left|\boldsymbol B\right|. $$ 事实上,使用类似的方法也可以得到 $$ \begin{aligned} \begin{vmatrix} \boldsymbol A&\boldsymbol C\\ \boldsymbol 0&\boldsymbol B \end{vmatrix}&=\left|\boldsymbol A\right|\left|\boldsymbol B\right|\\ \begin{vmatrix} \boldsymbol 0&\boldsymbol A\\ \boldsymbol B&\boldsymbol C \end{vmatrix}&=(-1)^{kr}\left|\boldsymbol A\right|\left|\boldsymbol B\right|. \end{aligned} $$ Laplace 定理还有其他应用,但仅就证明 Binet-Cauchy 这一点上,以上推论已足够。 ### 方阵乘积的行列式 接下来让我们转向对矩阵乘积的行列式的讨论。我们将证明 $\left|{\boldsymbol{AB}}\right|=\left|{\boldsymbol A}\right|\left|{\boldsymbol B}\right|$,也即如下推论。 $\bf Corollary\;4.\quad\left|{\boldsymbol{AB}}\right|=\left|{\boldsymbol A}\right|\left|{\boldsymbol B}\right|$. 等式右端使我们联想到行列式 $\begin{vmatrix} \boldsymbol A&\boldsymbol 0\\ \boldsymbol C&\boldsymbol B \end{vmatrix}$。我们应当使这个行列式能变换出 $\begin{vmatrix} \boldsymbol 0&\boldsymbol{AB}\\ \boldsymbol C&\boldsymbol B \end{vmatrix}$ 的形式,然后使用 Laplace 定理。方便起见可设 $\boldsymbol C=-\boldsymbol{I}$。为此考虑如下证法: $$ \begin{aligned} \left|{\boldsymbol A}\right|\left|{\boldsymbol B}\right|&=\begin{vmatrix}{\boldsymbol A}&{\bf 0}\\{-\boldsymbol{I}}&{\boldsymbol B}\end{vmatrix}= \begin{vmatrix} a_{11}&a_{12}&\cdots&a_{1n}&0&0&0&0\\ a_{21}&a_{22}&\cdots&a_{2n}&0&0&0&0\\ \vdots&\vdots&&\vdots&\vdots&\vdots&&\vdots\\ a_{n1}&a_{n2}&\cdots&a_{nn}&0&0&0&0\\ -1&0&0&0&b_{11}&b_{12}&\cdots&b_{1n}\\ 0&-1&0&0&b_{21}&b_{22}&\cdots&b_{2n}\\ \vdots&\vdots&&\vdots&\vdots&\vdots&&\vdots\\ 0&0&0&-1&b_{n1}&b_{n2}&\cdots&b_{nn}\\ \end{vmatrix}\\&= \begin{vmatrix} 0&0&0&0&\sum\limits_{k=1}^na_{1k}b_{k1}&\sum\limits_{k=1}^na_{1k}b_{k2}&\cdots&\sum\limits_{k=1}^na_{1k}b_{kn}\\ 0&0&0&0&\sum\limits_{k=1}^na_{2k}b_{k1}&\sum\limits_{k=1}^na_{2k}b_{k2}&\cdots&\sum\limits_{k=1}^na_{2k}b_{kn}\\ \vdots&\vdots&&\vdots&\vdots&\vdots&&\vdots\\ 0&0&0&0&\sum\limits_{k=1}^na_{nk}b_{k1}&\sum\limits_{k=1}^na_{nk}b_{k2}&\cdots&\sum\limits_{k=1}^na_{nk}b_{kn}\\ -1&0&0&0&b_{11}&b_{12}&\cdots&b_{1n}\\ 0&-1&0&0&b_{21}&b_{22}&\cdots&b_{2n}\\ \vdots&\vdots&&\vdots&\vdots&\vdots&&\vdots\\ 0&0&0&-1&b_{n1}&b_{n2}&\cdots&b_{nn}\\ \end{vmatrix}\\ &=\begin{vmatrix}{\bf 0}&{\boldsymbol{AB}}\\{\bf -I}&{\boldsymbol B}\end{vmatrix} \end{aligned} $$ 使用 Laplace 定理按其前 $n$ 行展开得 $$\begin{vmatrix}{\bf 0}&{\boldsymbol{AB}}\\-\boldsymbol{I}&{\boldsymbol B}\end{vmatrix}=\left|{\boldsymbol{AB}}\right|(-1)^{(1+\dots+n)+((n+1)+\dots+(n+n))}\left|-\boldsymbol{I}\right|=\left|{\boldsymbol{AB}}\right|.$$ 由此我们完成了证明。特别注意,以上矩阵 $\boldsymbol{A,B}$ 都是方阵。 ## Binet-Cauchy 公式的证明 现在我们可以来证明 Binet-Cauchy。注意到当 $s>n$ 时 $$ \operatorname{rank}(\boldsymbol{AB})\leqslant\operatorname{rank}(\boldsymbol{A})\leqslant n< s\,, $$ 于是此种情形下乘积不是满秩矩阵,其行列式为零。 以下考察 $s\leqslant n$ 时的情形。用两种方法计算下述行列式: $$ \boldsymbol D= \begin{vmatrix}{\boldsymbol A}&{\bf 0}\\{-\boldsymbol{I}}&{\boldsymbol B}\end{vmatrix} $$ 此处 $\boldsymbol{A}$ 为 $s\times n$ 的矩阵,而 $\boldsymbol{B}$ 是 $n\times s$。一方面,根据上一节的结论,它等于 $$ \begin{aligned} \begin{vmatrix}{\boldsymbol 0}&{\boldsymbol{AB}}\\{-\boldsymbol{I}}&{\boldsymbol B}\end{vmatrix}&=|\boldsymbol{AB}|(-1)^{(1+\cdots+s)+((n+1)+\cdots+(n+s))}|-\boldsymbol{I}|\\ &=(-1)^{sn}(-1)^n|\boldsymbol{AB}|\,. \end{aligned} $$ 另一方面,将其按前 $s$ 行展开,可得 $$ \boldsymbol{D}=\sum_{1\leqslant v_1<\cdots<v_s\leqslant n}\boldsymbol{A}\begin{pmatrix} 1,2,\cdots,s\\v_1,v_2,\cdots,v_s \end{pmatrix}(-1)^{(1+\cdots+s)+(v_1+\cdots+v_s)}\left|\left(\boldsymbol \varepsilon_{v'_1},\cdots,\boldsymbol \varepsilon_{v'_{n-s}},\boldsymbol B\right)\right|\,, $$ 其中 $\{v'_i\}_{1}^{n-s}$ 为 $[n]-\{v_1,\cdots,v_s\}$,且按递增顺序排列。将 $$ \left|\left(\boldsymbol \varepsilon_{v'_1},\cdots,\boldsymbol \varepsilon_{v'_{n-s}},\boldsymbol B\right)\right| $$ 按前 $n-s$ 列展开。显见前 $n-s$ 列只有一个子式非零,它是取 $v'_1,\cdots,v'_{n-s}$ 这几行构成的。因此 $$ \begin{aligned} &\left|\left(\boldsymbol \varepsilon_{v'_1},\cdots,\boldsymbol \varepsilon_{v'_{n-s}},\boldsymbol B\right)\right|\\ =&\;|-\boldsymbol I_{n-s}|(-1)^{\left(v'_1+\cdots+v'_{n-s}\right)+(1+\cdots+(n-s))}\boldsymbol B\begin{pmatrix} v_1,v_2,\cdots,v_s\\1,2,\cdots,s \end{pmatrix}\\ =&\;(-1)^{n-s}(-1)^{\left(v'_1+\cdots+v'_{n-s}\right)}(-1)^{\frac{1}{2}(n-s+1)(n-s)}\boldsymbol B\begin{pmatrix} v_1,v_2,\cdots,v_s\\1,2,\cdots,s \end{pmatrix}. \end{aligned} $$ 据此得 $$ \boldsymbol{D}=\sum_{1\leqslant v_1<\cdots<v_s\leqslant n}\boldsymbol{A}\begin{pmatrix} 1,2,\cdots,s\\v_1,v_2,\cdots,v_s \end{pmatrix}\cdot u\cdot \boldsymbol B\begin{pmatrix} v_1,v_2,\cdots,v_s\\1,2,\cdots,s \end{pmatrix}, $$ 其中 $$ \begin{aligned} u&=(-1)^{(1+\cdots+s)+(v_1+\cdots+v_s)}(-1)^{n-s}(-1)^{\left(v'_1+\cdots+v'_{n-s}\right)}(-1)^{\frac{1}{2}(n-s+1)(n-s)}\\ &=(-1)^{\frac{1}{2}s(s+1)}(-)^{\frac{1}{2}n(n+1)}(-1)^{n-s}(-1)^{\frac{1}{2}(n-s+1)(n-s)}\\ &=(-1)^{n^2+s^2-s(n+1)}\,. \end{aligned} $$ 对比两种求 $\boldsymbol D$ 的方法,可得 $$ \begin{aligned} (-1)^{sn}(-1)^n|\boldsymbol{AB}|&=\sum_{1\leqslant v_1<\cdots<v_s\leqslant n}\boldsymbol{A}\begin{pmatrix} 1,2,\cdots,s\\v_1,v_2,\cdots,v_s \end{pmatrix}(-1)^{n^2+s^2-s(n+1)}\boldsymbol B\begin{pmatrix} v_1,v_2,\cdots,v_s\\1,2,\cdots,s \end{pmatrix}\\ |\boldsymbol{AB}|&=(-1)^{(n-s)(n-s+1)}\sum_{1\leqslant v_1<\cdots<v_s\leqslant n}\begin{pmatrix} 1,2,\cdots,s\\v_1,v_2,\cdots,v_s \end{pmatrix} \boldsymbol B\begin{pmatrix} v_1,v_2,\cdots,v_s\\1,2,\cdots,s \end{pmatrix}\\ &=\sum_{1\leqslant v_1<\cdots<v_s\leqslant n}\begin{pmatrix} 1,2,\cdots,s\\v_1,v_2,\cdots,v_s \end{pmatrix} \boldsymbol B\begin{pmatrix} v_1,v_2,\cdots,v_s\\1,2,\cdots,s \end{pmatrix}. \end{aligned} $$ 至此我们完成了证明。 ------------ # 公式的经典应用 Binet-Cauchy 的应用场合颇为广泛。我们先以一个例子入手。 ${\bf Example.\quad}n$ 阶行列式 $$ \det\boldsymbol A:=\begin{vmatrix} a_1-b_1&a_1-b_2&\cdots&a_1-b_n\\ a_2-b_1&a_2-b_2&\cdots&a_2-b_n\\ \vdots&\vdots&\ddots&\vdots\\ a_n-b_1&a_n-b_2&\cdots&a_n-b_n\\ \end{vmatrix} $$ 的值可以用 Binet-Cauchy 公式求出。具体地,它等于 $$ \qquad\qquad\qquad\qquad\qquad\begin{aligned} \det\boldsymbol A&=\begin{vmatrix} \begin{pmatrix} a_1&-1\\ a_2&-1\\ \vdots&\vdots\\ a_n&-1 \end{pmatrix} \begin{pmatrix} 1&1&\cdots&1\\ b_1&b_2&\cdots&b_n \end{pmatrix} \end{vmatrix}\\ &=\begin{cases} 0,&n>2\\ (a_2-a_1)(b_2-b_1),&n=2\\ a_1-b_1,&n=1 \end{cases}\qquad\qquad\qquad\qquad\qquad\quad\blacksquare \end{aligned} $$ ## 积矩阵的各阶子式 Binet-Cauchy 公式的应用之一就是计算乘积矩阵的各阶子式。 $\bf Theorem\;5.\quad$设 $\boldsymbol{A}=(a_{ij})_{s\times n},\,\boldsymbol{B}=(b_{ij})_{n\times s}$,设 $r\leqslant s$,那么: - 若 $r>n$,则 $\boldsymbol{AB}$ 的 $r$ 阶子式为零。 - 若 $r\leqslant n$,则: $$ \boldsymbol{AB}\begin{pmatrix}i_1,i_2,\cdots,i_r\\j_1,j_2,\cdots,j_r\end{pmatrix}=\sum_{1\leqslant v_1<\cdots<v_r\leqslant n}\boldsymbol A\begin{pmatrix}i_1,i_2,\cdots,i_r\\v_1,v_2,\cdots,v_r\end{pmatrix}\boldsymbol B\begin{pmatrix}v_1,v_2,\cdots,v_r\\j_1,j_2,\cdots,j_r\end{pmatrix} $$ ### 定理的证明 $\boldsymbol{AB}$ 的任一 $r$ 阶子式 $\boldsymbol{AB}\begin{pmatrix}i_1,i_2,\cdots,i_r\\j_1,j_2,\cdots,j_r\end{pmatrix}$ 为 $$ \begin{aligned} \boldsymbol{AB}\begin{pmatrix}i_1,i_2,\cdots,i_r\\j_1,j_2,\cdots,j_r\end{pmatrix}&=\begin{vmatrix} \boldsymbol{AB}(i_1;j_1)&\boldsymbol{AB}(i_1;j_2)&\cdots&\boldsymbol{AB}(i_1;j_r)\\ \boldsymbol{AB}(i_2;j_1)&\boldsymbol{AB}(i_2;j_2)&\cdots&\boldsymbol{AB}(i_2;j_r)\\ \vdots&\vdots&\ddots&\vdots&\\ \boldsymbol{AB}(i_r;j_1)&\boldsymbol{AB}(i_r;j_2)&\cdots&\boldsymbol{AB}(i_r;j_r)\\ \end{vmatrix}\\ &=\begin{vmatrix} \begin{pmatrix} a_{i_1,1}&a_{i_1,2}&\cdots&a_{i_1,n}\\ a_{i_2,1}&a_{i_2,2}&\cdots&a_{i_2,n}\\ \vdots&\vdots&\ddots&\vdots\\ a_{i_r,1}&a_{i_r,2}&\cdots&a_{i_r,n} \end{pmatrix}\begin{pmatrix} b_{1,j_1}&b_{1,j_2}&\cdots&b_{1,j_r}\\ b_{2,j_1}&b_{2,j_2}&\cdots&b_{2,j_r}\\ \vdots&\vdots&\ddots&\vdots\\ b_{n,j_1}&b_{n,j_2}&\cdots&b_{n,j_r} \end{pmatrix} \end{vmatrix}. \end{aligned} $$ 利用 Binet-Cauchy 公式计算上述乘积矩阵的行列式,即可证明。 ### 应用举例 $\bf Example.\quad$设 $\boldsymbol A$ 为实数域上 $s\times n$ 矩阵,则 $\boldsymbol{AA}'$ 与 $\boldsymbol{A}'\boldsymbol A$ 的秩都等于 $\operatorname{rank}\boldsymbol A$。 无妨设 $\operatorname{rank}\boldsymbol A=r$,考虑 $\boldsymbol{AA}'$ 的任一 $r$ 阶主子式 $$ \begin{aligned} \boldsymbol{AA}'\begin{pmatrix} i_1,i_2,\cdots,i_r\\ i_1,i_2,\cdots,i_r \end{pmatrix}&=\sum_{1\leqslant v_1<\cdots<v_r\leqslant n}\boldsymbol A\begin{pmatrix} i_1,i_2,\cdots,i_r\\ v_1,v_2,\cdots,v_r \end{pmatrix}\boldsymbol A'\begin{pmatrix} v_1,v_2,\cdots,v_r\\ i_1,i_2,\cdots,i_r \end{pmatrix}\\ &=\sum_{1\leqslant v_1<\cdots<v_r\leqslant n}\Bigg[\boldsymbol A\begin{pmatrix} i_1,i_2,\cdots,i_r\\ v_1,v_2,\cdots,v_r \end{pmatrix}\Bigg]^2, \end{aligned} $$ 由于 $\operatorname{rank}\boldsymbol A=r$,因此存在 $\boldsymbol A$ 的一个 $r$ 阶子式非零,从而存在 $\boldsymbol{AA}'$ 的一个 $r$ 阶主子式非零,因此 $$ r\leqslant \operatorname{rank}\boldsymbol{AA}'\leqslant\operatorname{rank}\boldsymbol A=r\,. $$ 从而 $\operatorname{rank}\boldsymbol{AA}'=r$。类似地,$\operatorname{rank}\boldsymbol{A}'\boldsymbol A=r$。$\blacksquare$ $\bf Example.\quad$设 $\boldsymbol A$ 为 $n\times m$ 矩阵,且 $m\geqslant n-1$。则 $\boldsymbol{AA}'$ 的 $(1,1)$ 元代数余子式是 $$ (-1)^{1+1}\boldsymbol{AA}'\begin{pmatrix} 2,3,\cdots,n\\ 2,3,\cdots,n \end{pmatrix}=\sum_{1\leqslant v_1<\cdots<v_{n-1}\leqslant n}\Bigg[\boldsymbol{A}\begin{pmatrix} 2,3,\cdots,n\\ v_1,v_2,\cdots,v_{n-1} \end{pmatrix}\Bigg]^2 $$ 即 $\boldsymbol{AA}'$ 的 $(1,1)$ 元代数余子式是 $\boldsymbol{A}$ 的第一行元素的余子式的平方和。$\blacksquare$ ## 方阵乘积的秩与行列式 $\bf Example.\quad$设 $s_k=x_1^k+x_2^k+\cdots+x_n^k$,并设矩阵 $\boldsymbol A=(a_{ij})_{n\times n}$,其中 $$ a_{ij}=s_{i+j-2},\quad,i,j=1,2,\cdots,n. $$ 则有 $$ \begin{aligned} \det\boldsymbol A&=\begin{vmatrix} s_0&s_1&\cdots&s_{n-1}\\ s_1&s_2&\cdots&s_{n}\\ \vdots&\vdots&\ddots&\vdots\\ s_{n-1}&s_{n}&\cdots&s_{2n-2} \end{vmatrix}\\ &=\begin{vmatrix} \begin{pmatrix} 1&1&\cdots&1\\ x_1&x_2&\cdots&x_n\\ \vdots&\vdots&\ddots&\vdots\\ x^{n-1}_1&x^{n-1}_2&\cdots&x_n^{n-1} \end{pmatrix} \begin{pmatrix} 1&x_1&\cdots&x^{n-1}_1\\ 1&x_2&\cdots&x^{n-1}_2\\ \vdots&\vdots&\ddots&\vdots\\ 1&x_n&\cdots&x_n^{n-1} \end{pmatrix} \end{vmatrix}\\ &=\prod_{j<i}(x_i-x_j)^2\,.\;\;\,\blacksquare \end{aligned} $$ $\bf Example.\quad$设数域 $F$ 中有数列 $\{a\}$ 与 $\{c\}$,设 $$ u_t=\sum_{i=1}^nc_ia_i^t\,, $$ 并设矩阵 $$ \boldsymbol A=\begin{pmatrix} u_1&u_2&\cdots&u_n\\ u_2&u_3&\cdots&u_{n+1}\\ \vdots&\vdots&\ddots&\vdots\\ u_n&u_{n+1}&\cdots&u_{2n-1} \end{pmatrix}, $$ 现在来考虑方程 $$ \boldsymbol{Ax}=\boldsymbol B. $$ 考察其行列式,有 $$ \begin{aligned} \det\boldsymbol A&=\begin{vmatrix} \sum\limits_{i=1}^nc_ia_i&\sum\limits_{i=1}^nc_ia_i^{2}&\cdots&\sum\limits_{i=1}^nc_ia_i^n\\ \sum\limits_{i=1}^nc_ia_i^2&\sum\limits_{i=1}^nc_ia_i^{3}&\cdots&\sum\limits_{i=1}^nc_ia_i^{n+1}\\ \vdots&\vdots&\ddots&\vdots\\ \sum\limits_{i=1}^nc_ia_i^n&\sum\limits_{i=1}^nc_ia_i^{n+1}&\cdots&\sum\limits_{i=1}^nc_ia_i^{2n-1} \end{vmatrix}\\ &=\begin{vmatrix} \begin{pmatrix} c_1&c_2&\cdots&c_n\\ c_1a_1&c_2a_2&\cdots&c_na_n\\ \vdots&\vdots&\ddots&\vdots\\ c_1a_1^{n-1}&c_2a_2^{n-1}&\cdots&c_na_n^{n-1}\\ \end{pmatrix} \begin{pmatrix} a_1&a_1^2&\cdots&a^{n}_1\\ a_2&a_2^2&\cdots&a^{n}_2\\ \vdots&\vdots&\ddots&\vdots\\ a_n&a_n^2&\cdots&a_n^{n} \end{pmatrix} \end{vmatrix}\\ &=a_1c_1a_2c_2\cdots a_nc_n\prod_{j<i}(a_i-a_j)^2\,. \end{aligned} $$ 这表明方程 $\boldsymbol{Ax}=\boldsymbol B$ 有唯一解的充要条件是 $\{a\},\{c\}$ 全部非零,且 $\{a\}$ 两两不等。$\blacksquare$ ### 循环矩阵的行列式 所谓循环矩阵是以如下形式书写的矩阵: $$ {\boldsymbol A}=\sum_{k=1}^{n}a_{k}{\boldsymbol C}^{k-1} $$ 其中 $\boldsymbol C$ 是循环位移矩阵。 考虑商环 $\mathbb C[\![z]\!]\big/\!\left\langle z^n-1\right\rangle=\mathbb C[\![z]\!]\big/\!\left\langle\prod_{k<n}\left(z-\omega^k\right)\right\rangle$,其对应乘法为循环卷积。观察 $$ z\cdot f(z)=a_n+a_1z+a_2z^2+\cdots+a_{n-1}z^{n-1}, $$ 其与循环矩阵相邻两行间的形式如出一辙。为此考虑代入 $n$ 次单位根 $\omega$,就有 $$ \begin{aligned} |{\boldsymbol{ AB}}|&= \begin{vmatrix} f(1)&f(\omega)&f(\omega^{2})&\cdots&f(\omega^{n-1})\\ f(1)&\omega f(\omega)&\omega^2 f(\omega^{2})&\cdots&\omega^{n-1}f(\omega^{n-1})\\ \vdots&\vdots&\vdots&\ddots&\vdots\\ f(1)&\omega^{n-1}f(\omega)&\omega^{2(n-1)}f(\omega^{2})&\cdots&\omega^{(n-1)(n-1)}f(\omega^{n-1}) \end{vmatrix}\\ &=f(1)f(\omega)f(\omega^2)\cdots f(\omega^{n-1})\begin{vmatrix} 1&1&1&\cdots&1\\ 1&\omega&\omega^{2}&\cdots&\omega^{n-1}\\ 1&\omega^2&\omega^{4}&\cdots&\omega^{2(n-1)}\\ \vdots&\vdots&\vdots&\ddots&\vdots\\ 1&\omega^{n-1}&\omega^{2(n-1)}&\cdots&\omega^{(n-1)(n-1)} \end{vmatrix}, \end{aligned} $$ 由此便得到 $$ \det\mathbf A=\prod_{k=0}^{n-1}f(\omega^k)\,. $$ 此是逆用方阵乘积的行列式性质的一个例子。 $\bf Example.\quad$计算下面一类最简单的循环矩阵的行列式。 $$ \boldsymbol A=\begin{pmatrix} 1&2&3&\cdots&n-1&n\\ n&1&2&\cdots&n-2&n-1\\ n-1&n&1&\cdots&n-3&n-2\\ \vdots&\vdots&\vdots&\ddots&\vdots&\vdots\\ 3&4&5&\cdots&1&2\\ 2&3&4&\cdots&n&1 \end{pmatrix} $$ 代入公式得 $$ \det\boldsymbol A=\prod_{i=0}^{n-1}\sum_{j=1}^{n}j\omega^{i(j-1)}. $$ 注意到 $$ \sum_{k=1}^nkx^{k-1}=\frac{\text d}{\text dx}\frac{1-x^{n+1}}{1-x}=\frac{nx^{n+1}-(n+1)x^n+1}{(1-x)^2}\,, $$ 于是有 $$ \quad\qquad\qquad\qquad\begin{aligned} \det\boldsymbol A&=\frac{n(n+1)}{2}\prod_{i=1}^{n-1}\frac{n\!\left(\omega^i\right)^{n+1}-(n+1)\!\left(\omega^i\right)^n+1}{\left(1-\omega^i\right)^2}\\ &=\frac{n(n+1)}{2}\prod_{i=1}^{n-1}\frac{n\omega^i-n}{\left(1-\omega^i\right)^2}\\ &=\frac{n(n+1)}{2}\cdot(-1)^{n-1}n^{n-1}\cdot\prod_{0<i<n}\frac{1}{1-\omega^i}\\ &=(-1)^{n-1}\frac{n+1}{2}n^{n-1}\;. \qquad\qquad\qquad\qquad\qquad\qquad\quad\qquad\quad\blacksquare\end{aligned} $$ ## Cauchy 恒等式 $\;$ 与向量相关的不等式 Binet-Cauchy 公式的另一个重要应用是可以用以证明 Cauchy 恒等式一族,包括 Lagrange 恒等式等;亦可在此基础上证明与向量相关的一些不等式,例如 Cauchy 不等式。我们就以这项应用收尾。 $\bf Theorem\;6.\;(Cauchy)$ $$ \left(\sum_{i=1}^na_ic_i\right)\!\left(\sum_{i=1}^nb_id_i\right)- \left(\sum_{i=1}^na_id_i\right)\!\left(\sum_{i=1}^nb_ic_i\right)=\sum_{j<k}(a_jb_k-a_kb_j)(c_jd_k-c_kd_j) $$ ### 恒等式的证明 $$ \begin{aligned} \left(\sum_{i=1}^na_ic_i\right)&\!\left(\sum_{i=1}^nb_id_i\right)- \left(\sum_{i=1}^na_id_i\right)\!\left(\sum_{i=1}^nb_ic_i\right)\\ &=\begin{vmatrix} \sum\limits_{i=1}^na_ic_i&\sum\limits_{i=1}^na_id_i\\ \sum\limits_{i=1}^nb_ic_i&\sum\limits_{i=1}^nb_id_i \end{vmatrix}\\ &=\begin{vmatrix} \begin{pmatrix} a_1&a_2&\cdots&a_n\\ b_1&b_2&\cdots&b_n \end{pmatrix} \begin{pmatrix} c_1&d_1\\ c_2&d_2\\ \vdots&\vdots\\ c_n&d_n \end{pmatrix} \end{vmatrix}\\ &=\sum_{j<k}\begin{vmatrix}a_j&a_k\\b_j&b_k\end{vmatrix}\cdot\begin{vmatrix}c_j&d_j\\c_k&d_k\end{vmatrix}\\ &=\sum_{j<k}(a_jb_k-a_kb_j)(c_jd_k-c_kd_j) \end{aligned} $$ ### Lagrange 恒等式与 Cauchy 不等式 所谓 **Lagrange 恒等式**是指 $$ \left(\sum_{i=1}^na_i^2\right)\!\left(\sum_{i=1}^nb_i^2\right)- \left(\sum_{i=1}^na_ib_i\right)^2=\sum_{j<k}(a_jb_k-a_kb_j)^2. $$ 注意到若 $\boldsymbol a,\boldsymbol b$ 是二维向量,那么它有显然的几何意义 $$ |\boldsymbol a|^2\cdot|\boldsymbol b|^2-(\boldsymbol a\cdot \boldsymbol b)^2=|\boldsymbol a\times \boldsymbol b|^2, $$ 也即 $\left(\left|\boldsymbol a\right|\left|\boldsymbol b\right|\right)^2=\left(\left|\boldsymbol a\right|\left|\boldsymbol b\right|\sin\langle\boldsymbol a,\boldsymbol b\rangle\right)^2+\left(\left|\boldsymbol a\right|\left|\boldsymbol b\right|\cos\langle\boldsymbol a,\boldsymbol b\rangle\right)^2$。它的几何意义是相当明显的。 这个形式自然使我们联想到 Cuachy 不等式;事实上,上述对二维向量形式的讨论已经给出了一个二维的 Cauchy 不等式。现在让我们考察一般的 $n$ 维情形。 由 Lagrange 恒等式得 $$ \left(\sum_{i=1}^na_i^2\right)\!\left(\sum_{i=1}^nb_i^2\right)\geqslant \left(\sum_{i=1}^na_ib_i\right)^2, $$ 这就是著名的 **Cauchy 不等式**。等号成立当且仅当 $\forall j,k\,(j<k)$ 都有 $a_jb_k-a_kb_j=0$,即 $$ \operatorname{rank}\begin{pmatrix} a_1&a_2&\cdots&a_n\\ b_1&b_2&\cdots&b_n \end{pmatrix}\leqslant 1 $$ 也即 $n$ 维向量 $\boldsymbol a$ 与 $\boldsymbol b$ 线性相关。